Find the maximum area of a rectangle that can be drawn inside a circle's quarter












1












$begingroup$


How can I find the maximum area of the rectangle here? Given that the circle's radius is 6.
enter image description here



Any hint please?










share|cite|improve this question









$endgroup$












  • $begingroup$
    Maybe you can parametrise the rectangle? Maybe set the width of the rectangle to be $x$ and then calculate its area ...
    $endgroup$
    – Matti P.
    Jan 16 at 8:45










  • $begingroup$
    I tried, but I couldn't find any useful relationship between the width and the height.
    $endgroup$
    – Just_Cause
    Jan 16 at 8:49










  • $begingroup$
    Just make a big drawing, draw some auxiliary lines and see what you can come up with.
    $endgroup$
    – Matti P.
    Jan 16 at 9:01






  • 1




    $begingroup$
    I suspect the square with a vertex at the origin and opposite vertex at $(3sqrt{2},3sqrt{2}$)$ with area $18$ may be bigger
    $endgroup$
    – Henry
    Jan 16 at 9:14
















1












$begingroup$


How can I find the maximum area of the rectangle here? Given that the circle's radius is 6.
enter image description here



Any hint please?










share|cite|improve this question









$endgroup$












  • $begingroup$
    Maybe you can parametrise the rectangle? Maybe set the width of the rectangle to be $x$ and then calculate its area ...
    $endgroup$
    – Matti P.
    Jan 16 at 8:45










  • $begingroup$
    I tried, but I couldn't find any useful relationship between the width and the height.
    $endgroup$
    – Just_Cause
    Jan 16 at 8:49










  • $begingroup$
    Just make a big drawing, draw some auxiliary lines and see what you can come up with.
    $endgroup$
    – Matti P.
    Jan 16 at 9:01






  • 1




    $begingroup$
    I suspect the square with a vertex at the origin and opposite vertex at $(3sqrt{2},3sqrt{2}$)$ with area $18$ may be bigger
    $endgroup$
    – Henry
    Jan 16 at 9:14














1












1








1





$begingroup$


How can I find the maximum area of the rectangle here? Given that the circle's radius is 6.
enter image description here



Any hint please?










share|cite|improve this question









$endgroup$




How can I find the maximum area of the rectangle here? Given that the circle's radius is 6.
enter image description here



Any hint please?







calculus geometry optimization






share|cite|improve this question













share|cite|improve this question











share|cite|improve this question




share|cite|improve this question










asked Jan 16 at 8:39









Just_CauseJust_Cause

1257




1257












  • $begingroup$
    Maybe you can parametrise the rectangle? Maybe set the width of the rectangle to be $x$ and then calculate its area ...
    $endgroup$
    – Matti P.
    Jan 16 at 8:45










  • $begingroup$
    I tried, but I couldn't find any useful relationship between the width and the height.
    $endgroup$
    – Just_Cause
    Jan 16 at 8:49










  • $begingroup$
    Just make a big drawing, draw some auxiliary lines and see what you can come up with.
    $endgroup$
    – Matti P.
    Jan 16 at 9:01






  • 1




    $begingroup$
    I suspect the square with a vertex at the origin and opposite vertex at $(3sqrt{2},3sqrt{2}$)$ with area $18$ may be bigger
    $endgroup$
    – Henry
    Jan 16 at 9:14


















  • $begingroup$
    Maybe you can parametrise the rectangle? Maybe set the width of the rectangle to be $x$ and then calculate its area ...
    $endgroup$
    – Matti P.
    Jan 16 at 8:45










  • $begingroup$
    I tried, but I couldn't find any useful relationship between the width and the height.
    $endgroup$
    – Just_Cause
    Jan 16 at 8:49










  • $begingroup$
    Just make a big drawing, draw some auxiliary lines and see what you can come up with.
    $endgroup$
    – Matti P.
    Jan 16 at 9:01






  • 1




    $begingroup$
    I suspect the square with a vertex at the origin and opposite vertex at $(3sqrt{2},3sqrt{2}$)$ with area $18$ may be bigger
    $endgroup$
    – Henry
    Jan 16 at 9:14
















$begingroup$
Maybe you can parametrise the rectangle? Maybe set the width of the rectangle to be $x$ and then calculate its area ...
$endgroup$
– Matti P.
Jan 16 at 8:45




$begingroup$
Maybe you can parametrise the rectangle? Maybe set the width of the rectangle to be $x$ and then calculate its area ...
$endgroup$
– Matti P.
Jan 16 at 8:45












$begingroup$
I tried, but I couldn't find any useful relationship between the width and the height.
$endgroup$
– Just_Cause
Jan 16 at 8:49




$begingroup$
I tried, but I couldn't find any useful relationship between the width and the height.
$endgroup$
– Just_Cause
Jan 16 at 8:49












$begingroup$
Just make a big drawing, draw some auxiliary lines and see what you can come up with.
$endgroup$
– Matti P.
Jan 16 at 9:01




$begingroup$
Just make a big drawing, draw some auxiliary lines and see what you can come up with.
$endgroup$
– Matti P.
Jan 16 at 9:01




1




1




$begingroup$
I suspect the square with a vertex at the origin and opposite vertex at $(3sqrt{2},3sqrt{2}$)$ with area $18$ may be bigger
$endgroup$
– Henry
Jan 16 at 9:14




$begingroup$
I suspect the square with a vertex at the origin and opposite vertex at $(3sqrt{2},3sqrt{2}$)$ with area $18$ may be bigger
$endgroup$
– Henry
Jan 16 at 9:14










3 Answers
3






active

oldest

votes


















1












$begingroup$

Choosing one vertex to be $(r costheta, rsintheta)$ then another is $(r (costheta- sintheta),0)$, a third is $(0,r (costheta- sintheta))$ and the fourth is $(r sintheta, rcostheta)$



The area is then $2r^2 sintheta(costheta- sintheta) = sqrt{8}r^2 sin(theta) sinleft(fracpi4-thetaright) $ which is maximised when $theta= fracpi8$ with the maximum area being $r^2(sqrt{2}-1)$



Added to illustrate comment: If the arrangement is not symmetric, you will not get two vertices on the arc and two on the axes. Instead you will get coomething like this



enter image description here






share|cite|improve this answer











$endgroup$













  • $begingroup$
    Didn't get it how you assigned the coordinates of each vertex, can you explain please?
    $endgroup$
    – Just_Cause
    Jan 16 at 10:49










  • $begingroup$
    @Just_Cause - Are you happy with the $(r costheta, rsintheta)$ as a point on the circle? That lies on the line $y=x + r(cos theta - sin theta)$ and so the next vertex is where that line intersects $y=0$. The other two vertices are reflections in the line $y=x$
    $endgroup$
    – Henry
    Jan 16 at 13:20












  • $begingroup$
    Sure, but how did you assume that the small triangle near the x axis is isosceles? (45 45 90)? Doesn't this lead to the fact that the bigger triangle is isosceles too?
    $endgroup$
    – Just_Cause
    Jan 22 at 11:26










  • $begingroup$
    @Just_Cause If you do not have symmetry then either the vertices are not on the arc and axes or the quadrilateral is not a rectangle. So the triangle is isosceles
    $endgroup$
    – Henry
    Jan 22 at 12:49










  • $begingroup$
    Still don't get what has symmetry got to do here, do you mean the symmetry line of the triangle, you used it to divide the bigger triangle into two halves which would bisect the 90 angle?
    $endgroup$
    – Just_Cause
    Jan 27 at 21:00



















0












$begingroup$

I would compute the coordinates of the given points:
$$A(x_A,0),B(x_B,sqrt{6^2-x_B^2}),C(x_C,sqrt{6^2-x_C^2}),D(0,y_B)$$ Now it must be
$$vec{AB}=[x_B-x_A,sqrt{6^2-x_B^2}]$$ and $$vec{AD}=[-x_A,y_D]$$ and it must be
$$vec{AB}cdot vec{AD}=0$$
Can you proceed?






share|cite|improve this answer









$endgroup$













  • $begingroup$
    Appreciated, but I'm not really required to solve it using vectors.
    $endgroup$
    – Just_Cause
    Jan 16 at 10:44



















0












$begingroup$

Let $ABCD$ be our rectangle, where $A$ be placed on the perpendicular radius and $D$ be placed on the horizontal radius.



Also, let $OE$ be a radius of the circle such that $OEperp AD$, $OEcap BC={F}$, $OEcap AD={G}$ and let $HE$ be a diameter of the circle.



Now, since $G$ is a midpoint of $BC$, we obtain that $G$ is a midpoint of $AD$.



Thus, $OG=AG=DG$ and let $OG=x$, $DC=y$.



Now, since $measuredangle ECH=90^{circ},$ we obtain:
$$CE^2=EGcdot GH$$ or
$$x^2=(6-x-y)(6+x+y)$$ or
$$(x+y)^2=36-x^2$$ or
$$y=sqrt{36-x^2}-x.$$
Id est, $$S_{ABCD}=2xleft(sqrt{36-x^2}-xright).$$
Can you end it now?






share|cite|improve this answer









$endgroup$













  • $begingroup$
    Sure, but how did you get to $CE^2=EGcdot GH$ ?
    $endgroup$
    – Just_Cause
    Jan 16 at 10:44












  • $begingroup$
    Because the triangle CGH is a right-angled triangle and CE is an altitude of the triangle.
    $endgroup$
    – Michael Rozenberg
    Jan 16 at 11:32










  • $begingroup$
    Oh, i got it. One last thing that confuses me, is it permissible to assume that the radius is perpendicular on AD without an additional information from the question that denotes that? i.e OE bisects AD? As far as i know, i can't put any assumptions unless the question states a key info on some theorem
    $endgroup$
    – Just_Cause
    Jan 16 at 12:34








  • 1




    $begingroup$
    @Just_Cause If $OEperp AD$ then $OEperp BC$, which gives $BF=FC$ and from here $AG=GD.$
    $endgroup$
    – Michael Rozenberg
    Jan 16 at 13:23














Your Answer





StackExchange.ifUsing("editor", function () {
return StackExchange.using("mathjaxEditing", function () {
StackExchange.MarkdownEditor.creationCallbacks.add(function (editor, postfix) {
StackExchange.mathjaxEditing.prepareWmdForMathJax(editor, postfix, [["$", "$"], ["\\(","\\)"]]);
});
});
}, "mathjax-editing");

StackExchange.ready(function() {
var channelOptions = {
tags: "".split(" "),
id: "69"
};
initTagRenderer("".split(" "), "".split(" "), channelOptions);

StackExchange.using("externalEditor", function() {
// Have to fire editor after snippets, if snippets enabled
if (StackExchange.settings.snippets.snippetsEnabled) {
StackExchange.using("snippets", function() {
createEditor();
});
}
else {
createEditor();
}
});

function createEditor() {
StackExchange.prepareEditor({
heartbeatType: 'answer',
autoActivateHeartbeat: false,
convertImagesToLinks: true,
noModals: true,
showLowRepImageUploadWarning: true,
reputationToPostImages: 10,
bindNavPrevention: true,
postfix: "",
imageUploader: {
brandingHtml: "Powered by u003ca class="icon-imgur-white" href="https://imgur.com/"u003eu003c/au003e",
contentPolicyHtml: "User contributions licensed under u003ca href="https://creativecommons.org/licenses/by-sa/3.0/"u003ecc by-sa 3.0 with attribution requiredu003c/au003e u003ca href="https://stackoverflow.com/legal/content-policy"u003e(content policy)u003c/au003e",
allowUrls: true
},
noCode: true, onDemand: true,
discardSelector: ".discard-answer"
,immediatelyShowMarkdownHelp:true
});


}
});














draft saved

draft discarded


















StackExchange.ready(
function () {
StackExchange.openid.initPostLogin('.new-post-login', 'https%3a%2f%2fmath.stackexchange.com%2fquestions%2f3075481%2ffind-the-maximum-area-of-a-rectangle-that-can-be-drawn-inside-a-circles-quarter%23new-answer', 'question_page');
}
);

Post as a guest















Required, but never shown

























3 Answers
3






active

oldest

votes








3 Answers
3






active

oldest

votes









active

oldest

votes






active

oldest

votes









1












$begingroup$

Choosing one vertex to be $(r costheta, rsintheta)$ then another is $(r (costheta- sintheta),0)$, a third is $(0,r (costheta- sintheta))$ and the fourth is $(r sintheta, rcostheta)$



The area is then $2r^2 sintheta(costheta- sintheta) = sqrt{8}r^2 sin(theta) sinleft(fracpi4-thetaright) $ which is maximised when $theta= fracpi8$ with the maximum area being $r^2(sqrt{2}-1)$



Added to illustrate comment: If the arrangement is not symmetric, you will not get two vertices on the arc and two on the axes. Instead you will get coomething like this



enter image description here






share|cite|improve this answer











$endgroup$













  • $begingroup$
    Didn't get it how you assigned the coordinates of each vertex, can you explain please?
    $endgroup$
    – Just_Cause
    Jan 16 at 10:49










  • $begingroup$
    @Just_Cause - Are you happy with the $(r costheta, rsintheta)$ as a point on the circle? That lies on the line $y=x + r(cos theta - sin theta)$ and so the next vertex is where that line intersects $y=0$. The other two vertices are reflections in the line $y=x$
    $endgroup$
    – Henry
    Jan 16 at 13:20












  • $begingroup$
    Sure, but how did you assume that the small triangle near the x axis is isosceles? (45 45 90)? Doesn't this lead to the fact that the bigger triangle is isosceles too?
    $endgroup$
    – Just_Cause
    Jan 22 at 11:26










  • $begingroup$
    @Just_Cause If you do not have symmetry then either the vertices are not on the arc and axes or the quadrilateral is not a rectangle. So the triangle is isosceles
    $endgroup$
    – Henry
    Jan 22 at 12:49










  • $begingroup$
    Still don't get what has symmetry got to do here, do you mean the symmetry line of the triangle, you used it to divide the bigger triangle into two halves which would bisect the 90 angle?
    $endgroup$
    – Just_Cause
    Jan 27 at 21:00
















1












$begingroup$

Choosing one vertex to be $(r costheta, rsintheta)$ then another is $(r (costheta- sintheta),0)$, a third is $(0,r (costheta- sintheta))$ and the fourth is $(r sintheta, rcostheta)$



The area is then $2r^2 sintheta(costheta- sintheta) = sqrt{8}r^2 sin(theta) sinleft(fracpi4-thetaright) $ which is maximised when $theta= fracpi8$ with the maximum area being $r^2(sqrt{2}-1)$



Added to illustrate comment: If the arrangement is not symmetric, you will not get two vertices on the arc and two on the axes. Instead you will get coomething like this



enter image description here






share|cite|improve this answer











$endgroup$













  • $begingroup$
    Didn't get it how you assigned the coordinates of each vertex, can you explain please?
    $endgroup$
    – Just_Cause
    Jan 16 at 10:49










  • $begingroup$
    @Just_Cause - Are you happy with the $(r costheta, rsintheta)$ as a point on the circle? That lies on the line $y=x + r(cos theta - sin theta)$ and so the next vertex is where that line intersects $y=0$. The other two vertices are reflections in the line $y=x$
    $endgroup$
    – Henry
    Jan 16 at 13:20












  • $begingroup$
    Sure, but how did you assume that the small triangle near the x axis is isosceles? (45 45 90)? Doesn't this lead to the fact that the bigger triangle is isosceles too?
    $endgroup$
    – Just_Cause
    Jan 22 at 11:26










  • $begingroup$
    @Just_Cause If you do not have symmetry then either the vertices are not on the arc and axes or the quadrilateral is not a rectangle. So the triangle is isosceles
    $endgroup$
    – Henry
    Jan 22 at 12:49










  • $begingroup$
    Still don't get what has symmetry got to do here, do you mean the symmetry line of the triangle, you used it to divide the bigger triangle into two halves which would bisect the 90 angle?
    $endgroup$
    – Just_Cause
    Jan 27 at 21:00














1












1








1





$begingroup$

Choosing one vertex to be $(r costheta, rsintheta)$ then another is $(r (costheta- sintheta),0)$, a third is $(0,r (costheta- sintheta))$ and the fourth is $(r sintheta, rcostheta)$



The area is then $2r^2 sintheta(costheta- sintheta) = sqrt{8}r^2 sin(theta) sinleft(fracpi4-thetaright) $ which is maximised when $theta= fracpi8$ with the maximum area being $r^2(sqrt{2}-1)$



Added to illustrate comment: If the arrangement is not symmetric, you will not get two vertices on the arc and two on the axes. Instead you will get coomething like this



enter image description here






share|cite|improve this answer











$endgroup$



Choosing one vertex to be $(r costheta, rsintheta)$ then another is $(r (costheta- sintheta),0)$, a third is $(0,r (costheta- sintheta))$ and the fourth is $(r sintheta, rcostheta)$



The area is then $2r^2 sintheta(costheta- sintheta) = sqrt{8}r^2 sin(theta) sinleft(fracpi4-thetaright) $ which is maximised when $theta= fracpi8$ with the maximum area being $r^2(sqrt{2}-1)$



Added to illustrate comment: If the arrangement is not symmetric, you will not get two vertices on the arc and two on the axes. Instead you will get coomething like this



enter image description here







share|cite|improve this answer














share|cite|improve this answer



share|cite|improve this answer








edited Jan 28 at 1:15

























answered Jan 16 at 9:12









HenryHenry

101k482169




101k482169












  • $begingroup$
    Didn't get it how you assigned the coordinates of each vertex, can you explain please?
    $endgroup$
    – Just_Cause
    Jan 16 at 10:49










  • $begingroup$
    @Just_Cause - Are you happy with the $(r costheta, rsintheta)$ as a point on the circle? That lies on the line $y=x + r(cos theta - sin theta)$ and so the next vertex is where that line intersects $y=0$. The other two vertices are reflections in the line $y=x$
    $endgroup$
    – Henry
    Jan 16 at 13:20












  • $begingroup$
    Sure, but how did you assume that the small triangle near the x axis is isosceles? (45 45 90)? Doesn't this lead to the fact that the bigger triangle is isosceles too?
    $endgroup$
    – Just_Cause
    Jan 22 at 11:26










  • $begingroup$
    @Just_Cause If you do not have symmetry then either the vertices are not on the arc and axes or the quadrilateral is not a rectangle. So the triangle is isosceles
    $endgroup$
    – Henry
    Jan 22 at 12:49










  • $begingroup$
    Still don't get what has symmetry got to do here, do you mean the symmetry line of the triangle, you used it to divide the bigger triangle into two halves which would bisect the 90 angle?
    $endgroup$
    – Just_Cause
    Jan 27 at 21:00


















  • $begingroup$
    Didn't get it how you assigned the coordinates of each vertex, can you explain please?
    $endgroup$
    – Just_Cause
    Jan 16 at 10:49










  • $begingroup$
    @Just_Cause - Are you happy with the $(r costheta, rsintheta)$ as a point on the circle? That lies on the line $y=x + r(cos theta - sin theta)$ and so the next vertex is where that line intersects $y=0$. The other two vertices are reflections in the line $y=x$
    $endgroup$
    – Henry
    Jan 16 at 13:20












  • $begingroup$
    Sure, but how did you assume that the small triangle near the x axis is isosceles? (45 45 90)? Doesn't this lead to the fact that the bigger triangle is isosceles too?
    $endgroup$
    – Just_Cause
    Jan 22 at 11:26










  • $begingroup$
    @Just_Cause If you do not have symmetry then either the vertices are not on the arc and axes or the quadrilateral is not a rectangle. So the triangle is isosceles
    $endgroup$
    – Henry
    Jan 22 at 12:49










  • $begingroup$
    Still don't get what has symmetry got to do here, do you mean the symmetry line of the triangle, you used it to divide the bigger triangle into two halves which would bisect the 90 angle?
    $endgroup$
    – Just_Cause
    Jan 27 at 21:00
















$begingroup$
Didn't get it how you assigned the coordinates of each vertex, can you explain please?
$endgroup$
– Just_Cause
Jan 16 at 10:49




$begingroup$
Didn't get it how you assigned the coordinates of each vertex, can you explain please?
$endgroup$
– Just_Cause
Jan 16 at 10:49












$begingroup$
@Just_Cause - Are you happy with the $(r costheta, rsintheta)$ as a point on the circle? That lies on the line $y=x + r(cos theta - sin theta)$ and so the next vertex is where that line intersects $y=0$. The other two vertices are reflections in the line $y=x$
$endgroup$
– Henry
Jan 16 at 13:20






$begingroup$
@Just_Cause - Are you happy with the $(r costheta, rsintheta)$ as a point on the circle? That lies on the line $y=x + r(cos theta - sin theta)$ and so the next vertex is where that line intersects $y=0$. The other two vertices are reflections in the line $y=x$
$endgroup$
– Henry
Jan 16 at 13:20














$begingroup$
Sure, but how did you assume that the small triangle near the x axis is isosceles? (45 45 90)? Doesn't this lead to the fact that the bigger triangle is isosceles too?
$endgroup$
– Just_Cause
Jan 22 at 11:26




$begingroup$
Sure, but how did you assume that the small triangle near the x axis is isosceles? (45 45 90)? Doesn't this lead to the fact that the bigger triangle is isosceles too?
$endgroup$
– Just_Cause
Jan 22 at 11:26












$begingroup$
@Just_Cause If you do not have symmetry then either the vertices are not on the arc and axes or the quadrilateral is not a rectangle. So the triangle is isosceles
$endgroup$
– Henry
Jan 22 at 12:49




$begingroup$
@Just_Cause If you do not have symmetry then either the vertices are not on the arc and axes or the quadrilateral is not a rectangle. So the triangle is isosceles
$endgroup$
– Henry
Jan 22 at 12:49












$begingroup$
Still don't get what has symmetry got to do here, do you mean the symmetry line of the triangle, you used it to divide the bigger triangle into two halves which would bisect the 90 angle?
$endgroup$
– Just_Cause
Jan 27 at 21:00




$begingroup$
Still don't get what has symmetry got to do here, do you mean the symmetry line of the triangle, you used it to divide the bigger triangle into two halves which would bisect the 90 angle?
$endgroup$
– Just_Cause
Jan 27 at 21:00











0












$begingroup$

I would compute the coordinates of the given points:
$$A(x_A,0),B(x_B,sqrt{6^2-x_B^2}),C(x_C,sqrt{6^2-x_C^2}),D(0,y_B)$$ Now it must be
$$vec{AB}=[x_B-x_A,sqrt{6^2-x_B^2}]$$ and $$vec{AD}=[-x_A,y_D]$$ and it must be
$$vec{AB}cdot vec{AD}=0$$
Can you proceed?






share|cite|improve this answer









$endgroup$













  • $begingroup$
    Appreciated, but I'm not really required to solve it using vectors.
    $endgroup$
    – Just_Cause
    Jan 16 at 10:44
















0












$begingroup$

I would compute the coordinates of the given points:
$$A(x_A,0),B(x_B,sqrt{6^2-x_B^2}),C(x_C,sqrt{6^2-x_C^2}),D(0,y_B)$$ Now it must be
$$vec{AB}=[x_B-x_A,sqrt{6^2-x_B^2}]$$ and $$vec{AD}=[-x_A,y_D]$$ and it must be
$$vec{AB}cdot vec{AD}=0$$
Can you proceed?






share|cite|improve this answer









$endgroup$













  • $begingroup$
    Appreciated, but I'm not really required to solve it using vectors.
    $endgroup$
    – Just_Cause
    Jan 16 at 10:44














0












0








0





$begingroup$

I would compute the coordinates of the given points:
$$A(x_A,0),B(x_B,sqrt{6^2-x_B^2}),C(x_C,sqrt{6^2-x_C^2}),D(0,y_B)$$ Now it must be
$$vec{AB}=[x_B-x_A,sqrt{6^2-x_B^2}]$$ and $$vec{AD}=[-x_A,y_D]$$ and it must be
$$vec{AB}cdot vec{AD}=0$$
Can you proceed?






share|cite|improve this answer









$endgroup$



I would compute the coordinates of the given points:
$$A(x_A,0),B(x_B,sqrt{6^2-x_B^2}),C(x_C,sqrt{6^2-x_C^2}),D(0,y_B)$$ Now it must be
$$vec{AB}=[x_B-x_A,sqrt{6^2-x_B^2}]$$ and $$vec{AD}=[-x_A,y_D]$$ and it must be
$$vec{AB}cdot vec{AD}=0$$
Can you proceed?







share|cite|improve this answer












share|cite|improve this answer



share|cite|improve this answer










answered Jan 16 at 8:55









Dr. Sonnhard GraubnerDr. Sonnhard Graubner

78.4k42867




78.4k42867












  • $begingroup$
    Appreciated, but I'm not really required to solve it using vectors.
    $endgroup$
    – Just_Cause
    Jan 16 at 10:44


















  • $begingroup$
    Appreciated, but I'm not really required to solve it using vectors.
    $endgroup$
    – Just_Cause
    Jan 16 at 10:44
















$begingroup$
Appreciated, but I'm not really required to solve it using vectors.
$endgroup$
– Just_Cause
Jan 16 at 10:44




$begingroup$
Appreciated, but I'm not really required to solve it using vectors.
$endgroup$
– Just_Cause
Jan 16 at 10:44











0












$begingroup$

Let $ABCD$ be our rectangle, where $A$ be placed on the perpendicular radius and $D$ be placed on the horizontal radius.



Also, let $OE$ be a radius of the circle such that $OEperp AD$, $OEcap BC={F}$, $OEcap AD={G}$ and let $HE$ be a diameter of the circle.



Now, since $G$ is a midpoint of $BC$, we obtain that $G$ is a midpoint of $AD$.



Thus, $OG=AG=DG$ and let $OG=x$, $DC=y$.



Now, since $measuredangle ECH=90^{circ},$ we obtain:
$$CE^2=EGcdot GH$$ or
$$x^2=(6-x-y)(6+x+y)$$ or
$$(x+y)^2=36-x^2$$ or
$$y=sqrt{36-x^2}-x.$$
Id est, $$S_{ABCD}=2xleft(sqrt{36-x^2}-xright).$$
Can you end it now?






share|cite|improve this answer









$endgroup$













  • $begingroup$
    Sure, but how did you get to $CE^2=EGcdot GH$ ?
    $endgroup$
    – Just_Cause
    Jan 16 at 10:44












  • $begingroup$
    Because the triangle CGH is a right-angled triangle and CE is an altitude of the triangle.
    $endgroup$
    – Michael Rozenberg
    Jan 16 at 11:32










  • $begingroup$
    Oh, i got it. One last thing that confuses me, is it permissible to assume that the radius is perpendicular on AD without an additional information from the question that denotes that? i.e OE bisects AD? As far as i know, i can't put any assumptions unless the question states a key info on some theorem
    $endgroup$
    – Just_Cause
    Jan 16 at 12:34








  • 1




    $begingroup$
    @Just_Cause If $OEperp AD$ then $OEperp BC$, which gives $BF=FC$ and from here $AG=GD.$
    $endgroup$
    – Michael Rozenberg
    Jan 16 at 13:23


















0












$begingroup$

Let $ABCD$ be our rectangle, where $A$ be placed on the perpendicular radius and $D$ be placed on the horizontal radius.



Also, let $OE$ be a radius of the circle such that $OEperp AD$, $OEcap BC={F}$, $OEcap AD={G}$ and let $HE$ be a diameter of the circle.



Now, since $G$ is a midpoint of $BC$, we obtain that $G$ is a midpoint of $AD$.



Thus, $OG=AG=DG$ and let $OG=x$, $DC=y$.



Now, since $measuredangle ECH=90^{circ},$ we obtain:
$$CE^2=EGcdot GH$$ or
$$x^2=(6-x-y)(6+x+y)$$ or
$$(x+y)^2=36-x^2$$ or
$$y=sqrt{36-x^2}-x.$$
Id est, $$S_{ABCD}=2xleft(sqrt{36-x^2}-xright).$$
Can you end it now?






share|cite|improve this answer









$endgroup$













  • $begingroup$
    Sure, but how did you get to $CE^2=EGcdot GH$ ?
    $endgroup$
    – Just_Cause
    Jan 16 at 10:44












  • $begingroup$
    Because the triangle CGH is a right-angled triangle and CE is an altitude of the triangle.
    $endgroup$
    – Michael Rozenberg
    Jan 16 at 11:32










  • $begingroup$
    Oh, i got it. One last thing that confuses me, is it permissible to assume that the radius is perpendicular on AD without an additional information from the question that denotes that? i.e OE bisects AD? As far as i know, i can't put any assumptions unless the question states a key info on some theorem
    $endgroup$
    – Just_Cause
    Jan 16 at 12:34








  • 1




    $begingroup$
    @Just_Cause If $OEperp AD$ then $OEperp BC$, which gives $BF=FC$ and from here $AG=GD.$
    $endgroup$
    – Michael Rozenberg
    Jan 16 at 13:23
















0












0








0





$begingroup$

Let $ABCD$ be our rectangle, where $A$ be placed on the perpendicular radius and $D$ be placed on the horizontal radius.



Also, let $OE$ be a radius of the circle such that $OEperp AD$, $OEcap BC={F}$, $OEcap AD={G}$ and let $HE$ be a diameter of the circle.



Now, since $G$ is a midpoint of $BC$, we obtain that $G$ is a midpoint of $AD$.



Thus, $OG=AG=DG$ and let $OG=x$, $DC=y$.



Now, since $measuredangle ECH=90^{circ},$ we obtain:
$$CE^2=EGcdot GH$$ or
$$x^2=(6-x-y)(6+x+y)$$ or
$$(x+y)^2=36-x^2$$ or
$$y=sqrt{36-x^2}-x.$$
Id est, $$S_{ABCD}=2xleft(sqrt{36-x^2}-xright).$$
Can you end it now?






share|cite|improve this answer









$endgroup$



Let $ABCD$ be our rectangle, where $A$ be placed on the perpendicular radius and $D$ be placed on the horizontal radius.



Also, let $OE$ be a radius of the circle such that $OEperp AD$, $OEcap BC={F}$, $OEcap AD={G}$ and let $HE$ be a diameter of the circle.



Now, since $G$ is a midpoint of $BC$, we obtain that $G$ is a midpoint of $AD$.



Thus, $OG=AG=DG$ and let $OG=x$, $DC=y$.



Now, since $measuredangle ECH=90^{circ},$ we obtain:
$$CE^2=EGcdot GH$$ or
$$x^2=(6-x-y)(6+x+y)$$ or
$$(x+y)^2=36-x^2$$ or
$$y=sqrt{36-x^2}-x.$$
Id est, $$S_{ABCD}=2xleft(sqrt{36-x^2}-xright).$$
Can you end it now?







share|cite|improve this answer












share|cite|improve this answer



share|cite|improve this answer










answered Jan 16 at 9:15









Michael RozenbergMichael Rozenberg

109k1896201




109k1896201












  • $begingroup$
    Sure, but how did you get to $CE^2=EGcdot GH$ ?
    $endgroup$
    – Just_Cause
    Jan 16 at 10:44












  • $begingroup$
    Because the triangle CGH is a right-angled triangle and CE is an altitude of the triangle.
    $endgroup$
    – Michael Rozenberg
    Jan 16 at 11:32










  • $begingroup$
    Oh, i got it. One last thing that confuses me, is it permissible to assume that the radius is perpendicular on AD without an additional information from the question that denotes that? i.e OE bisects AD? As far as i know, i can't put any assumptions unless the question states a key info on some theorem
    $endgroup$
    – Just_Cause
    Jan 16 at 12:34








  • 1




    $begingroup$
    @Just_Cause If $OEperp AD$ then $OEperp BC$, which gives $BF=FC$ and from here $AG=GD.$
    $endgroup$
    – Michael Rozenberg
    Jan 16 at 13:23




















  • $begingroup$
    Sure, but how did you get to $CE^2=EGcdot GH$ ?
    $endgroup$
    – Just_Cause
    Jan 16 at 10:44












  • $begingroup$
    Because the triangle CGH is a right-angled triangle and CE is an altitude of the triangle.
    $endgroup$
    – Michael Rozenberg
    Jan 16 at 11:32










  • $begingroup$
    Oh, i got it. One last thing that confuses me, is it permissible to assume that the radius is perpendicular on AD without an additional information from the question that denotes that? i.e OE bisects AD? As far as i know, i can't put any assumptions unless the question states a key info on some theorem
    $endgroup$
    – Just_Cause
    Jan 16 at 12:34








  • 1




    $begingroup$
    @Just_Cause If $OEperp AD$ then $OEperp BC$, which gives $BF=FC$ and from here $AG=GD.$
    $endgroup$
    – Michael Rozenberg
    Jan 16 at 13:23


















$begingroup$
Sure, but how did you get to $CE^2=EGcdot GH$ ?
$endgroup$
– Just_Cause
Jan 16 at 10:44






$begingroup$
Sure, but how did you get to $CE^2=EGcdot GH$ ?
$endgroup$
– Just_Cause
Jan 16 at 10:44














$begingroup$
Because the triangle CGH is a right-angled triangle and CE is an altitude of the triangle.
$endgroup$
– Michael Rozenberg
Jan 16 at 11:32




$begingroup$
Because the triangle CGH is a right-angled triangle and CE is an altitude of the triangle.
$endgroup$
– Michael Rozenberg
Jan 16 at 11:32












$begingroup$
Oh, i got it. One last thing that confuses me, is it permissible to assume that the radius is perpendicular on AD without an additional information from the question that denotes that? i.e OE bisects AD? As far as i know, i can't put any assumptions unless the question states a key info on some theorem
$endgroup$
– Just_Cause
Jan 16 at 12:34






$begingroup$
Oh, i got it. One last thing that confuses me, is it permissible to assume that the radius is perpendicular on AD without an additional information from the question that denotes that? i.e OE bisects AD? As far as i know, i can't put any assumptions unless the question states a key info on some theorem
$endgroup$
– Just_Cause
Jan 16 at 12:34






1




1




$begingroup$
@Just_Cause If $OEperp AD$ then $OEperp BC$, which gives $BF=FC$ and from here $AG=GD.$
$endgroup$
– Michael Rozenberg
Jan 16 at 13:23






$begingroup$
@Just_Cause If $OEperp AD$ then $OEperp BC$, which gives $BF=FC$ and from here $AG=GD.$
$endgroup$
– Michael Rozenberg
Jan 16 at 13:23




















draft saved

draft discarded




















































Thanks for contributing an answer to Mathematics Stack Exchange!


  • Please be sure to answer the question. Provide details and share your research!

But avoid



  • Asking for help, clarification, or responding to other answers.

  • Making statements based on opinion; back them up with references or personal experience.


Use MathJax to format equations. MathJax reference.


To learn more, see our tips on writing great answers.




draft saved


draft discarded














StackExchange.ready(
function () {
StackExchange.openid.initPostLogin('.new-post-login', 'https%3a%2f%2fmath.stackexchange.com%2fquestions%2f3075481%2ffind-the-maximum-area-of-a-rectangle-that-can-be-drawn-inside-a-circles-quarter%23new-answer', 'question_page');
}
);

Post as a guest















Required, but never shown





















































Required, but never shown














Required, but never shown












Required, but never shown







Required, but never shown

































Required, but never shown














Required, but never shown












Required, but never shown







Required, but never shown







Popular posts from this blog

Human spaceflight

Can not write log (Is /dev/pts mounted?) - openpty in Ubuntu-on-Windows?

File:DeusFollowingSea.jpg